LSAT and Law School Admissions Forum

Get expert LSAT preparation and law school admissions advice from PowerScore Test Preparation.

 Administrator
PowerScore Staff
  • PowerScore Staff
  • Posts: 8937
  • Joined: Feb 02, 2011
|
#33618
Complete Question Explanation
(See the complete passage discussion here: lsat/viewtopic.php?t=13783)

The correct answer choice is (D)

Approach this Weaken question as you would any similar Logical Reasoning question: the correct answer choice must undermine the conclusion of the argument outlined in the first paragraph of passage B. The author explains her opposition to software patents, arguing that such patents impede innovation:
  • Cause ..... ..... ..... ..... ..... Effect

    Software patents ..... :arrow: ..... Innovation
The correct answer choice would weaken this conclusion by showing that software patents do not necessarily impede, and might actually promote, innovation.

Answer choice (A): This is an attractive answer choice, because if software patents expire after 20 years, their negative effects would not persist interminably. However, just because the effects are limited in time does not make them any less severe. Inhibiting innovation for 20 years may still present an astronomical opportunity cost. This answer choice is incorrect.

Answer choice (B): This is the Opposite answer. If open-source software were more reliable than patented software, this would strengthen the author’s opposition to software patents.

Answer choice (C): Just because some companies oppose software patents for self-interested reasons does not make their argument any less credible. This is a classic Source Argument flaw: we cannot weaken an argument by questioning the character or motivations of those making the argument. This answer choice is incorrect.

Answer choice (D): This is the correct answer choice. If innovation becomes less profitable as a result of not patenting software, vendors would have little incentive to continue innovating. Innovation would be inhibited not by software patents (as the author suggests), but rather by the absence of patents. This would weaken the author’s argument, and challenge the defense of open-source software outlined in the first paragraph of passage.

Answer choice (E): Who benefits from software innovations has no bearing on whether patented software impedes innovation. This answer choice is incorrect.
 avengingangel
  • Posts: 275
  • Joined: Jun 14, 2016
|
#30916
Why is D the correct answer? The first paragraph of Passage B does not talk about profit or money at all... It talks about impeding innovation, which is why I chose E. Thanks!
 David Boyle
PowerScore Staff
  • PowerScore Staff
  • Posts: 836
  • Joined: Jun 07, 2013
|
#31301
avengingangel wrote:Why is D the correct answer? The first paragraph of Passage B does not talk about profit or money at all... It talks about impeding innovation, which is why I chose E. Thanks!

Hello,

"We believe such massive software patent portfolios are ripe for misuse because of the questionable nature of many software patents generally and because of the high cost of patent litigation." gets mentioned in at least the latter part of passage B. So, money is germane.
Then, as for answer D, "Software innovation would be less profitable if software could not be patented.": in the contrapositive, it's like

profit :arrow: patents.

That disagrees with the first part of passage B, "Software makers like ours have consistently taken the position that patents generally impede innovation in software development and are inconsistent with open-source/free software." So answer D is correct.

David
 mN2mmvf
  • Posts: 113
  • Joined: Jul 06, 2017
|
#37204
I agree with the confusion about choice (D). Companies do not need to be profitable in order to be innovative, and there's no clear relationship stated between greater profits and a larger R&D budget, or lower profits and a smaller R&D budget. That's why I chose answer (A) instead: it doesn't cast a lot of doubt, but it casts some. We know that our objective is to demonstrate that patents and innovation can coexist happily, and the existence of a time-limit on patents seems to suggest that they can.
 nicholaspavic
PowerScore Staff
  • PowerScore Staff
  • Posts: 271
  • Joined: Jun 12, 2017
|
#37617
I chose answer (A) instead: it doesn't cast a lot of doubt, but it casts some.
Hi mn2,

This is a very attactive wrong answer choice. And certainly, it would seem that some doubt is being cast upon the the concept of:

Software patents :arrow: Innovation

However, does Answer Option (A) truly attack the sufficient here? In other words, does the time limit truly eliminate the sufficient condition of software patents? In fact, it does not. Let's assume that instead of 20 years, you had a situation where software patents only lasted a month. Could software patents STILL limit the crucial advances made in my favorite app? In fact, it could. It could limit those advances in a way quite material to me. Perhaps Angry Birds is unplayable for a month because of the lack of advances. (Perish the thought!) And in fact, that shortening of the time limit still demonstrates the validity of the author's argument. Thus, Answer Option (A) does attack the sufficient condition whereas Answer (D) does.

This is a very tricky concept, but if you find yourself still getting caught up in it, try playing with the time limit suggested in the answer choice and testing it against the conditional reasoning offered in the stimulus. Make that time limit very short or very long and ask yourself: Does the logic hold? That may help you visualize the actual parameters of the conditional logic offered by the author.

Thanks for the great question and please know, you are on the right track. Your comment shows me that you are considering the arguments deeply and thoughtfully.
 cmorris32
  • Posts: 92
  • Joined: May 05, 2020
|
#77650
Hello! I am really struggling to see how answer choice D weakens the corporation’s argument in the first paragraph of Passage B. I don’t see how less profit gives reason to suppose that there would be a decreased incentive to innovate. It seems like the answer choice is saying: NOT software patented → less profit → decreased incentive to innovate. However, I don’t see how we can assume that less profit means a decreased incentive to innovate. What if the profit went from $2million to $1.5million? Clearly that is still a huge incentive to innovate. It feels like I am expecting Answer Choice D to weaken the argument more than it does. Maybe D is right because it gives me the slightest (maybe like 2%) reason to suppose that software patents impede on innovation? Thanks for any help in advance!
 golions
  • Posts: 1
  • Joined: Aug 10, 2020
|
#77926
Another question about this:

In my view, the position taken in the first paragraph is NOT just software patents :arrow: ~innovation. Rather it is software patents :arrow: ~innovation AND "inconsistent with open-source/free software". So if we wanted to weaken this conditional, we can show that patents does not harm innovation OR that patents are not inconsistent with open-source software OR the patents are not inconsistent with free software. In my reading, we now have three pathways to weaken the claim, instead of just the first that has been discussed so far.

In light of this, here is how I read A and D (the two I had narrowed it down to):

(A): Seems right to me, if only because it is plausible, and all the others are shaky. If patents expire in 20 years or less (or in principal, if they expire at all) then the software is now open-source (and to the extent that it can be now made by anyone, which usually though not necessarily leads to software becoming free, it is free). This may not seem intuitive because we think of open-source as being something that was accessible from the beginning - but open-source is never defined as such, and it seems plausible to say that open-source is just the absence of a patent at some point. Secondly, I am assuming that when patents are lifted the thing becomes free.

(D) Who cares if software innovation is less profitable without patents? Software companies might - but the open-source companies themselves already are giving software away for free, ie no profits (as is their principal). So I would think to the extent that Passage B believes that their 2/3 of their deeply held principles (software innovation and free software) are not in contradiction, they would say that software innovation does not depend on profits. Put another way, saying that this weakens their argument is almost to ignore their position in the first place.

But even if we're not convinced of the above, why should profits drive innovation? It seems intuitive, but the LSATs' bread and butter is getting you to pick the intuitive answer that is not actually logically airtight or requires us to smuggle in that intuition as given. My LSAT radar immediately really disliked this answer for this exact reason. And if we can smuggle in profits drive innovation, why not also smuggle in expiring patents :arrow: open-source software (or free software)?
 Rachael Wilkenfeld
PowerScore Staff
  • PowerScore Staff
  • Posts: 1419
  • Joined: Dec 15, 2011
|
#77991
cmorris, I'll take your question first, both because you asked first, and because your answer is more straightforward. An answer choice will either weaken or it won't. if you can see that it weakens even 2%, you've found your answer. Here, answer choice (D) gives us a reason to think that patents don't hurt innovation---patents lead to profit. While we shouldn't use outside information, the idea that money can be a motivator is something basic enough to assume.

Now, golions, on to your reading of this passage. It's fair to say that patents :arrow: inconsistent with open source/free software. I don't know you can take the second step of separating open source/free software, as the author seems to be using them as a package deal. We can't make the leap you make for answer choice (A). Think, for example, about Microsoft Office. Just because it may not be patented (there are several open source and free alternatives) doesn't mean that the Office software becomes automatically free/open source. People still pay for Office; Microsoft didn't release the code at any point. Those who created open source software were just able to engineer it themselves. So even after the patent expires, the specific software doesn't automatically become free/open source. There's nothing in the passage to suggest that's how it would work.

For answer choice (D), we would weaken the argument as described above. Money is a clear motivator for the author of passage B. They specifically state that they are taking an action (patenting their products) for a financial reason. The author links money and motivation himself as part of passage B.

Hope that helps both of you,
Rachael
User avatar
 Albertlyu
  • Posts: 98
  • Joined: Jul 18, 2020
|
#81178
Rachael Wilkenfeld wrote:cmorris, I'll take your question first, both because you asked first, and because your answer is more straightforward. An answer choice will either weaken or it won't. if you can see that it weakens even 2%, you've found your answer. Here, answer choice (D) gives us a reason to think that patents don't hurt innovation---patents lead to profit. While we shouldn't use outside information, the idea that money can be a motivator is something basic enough to assume.

Now, golions, on to your reading of this passage. It's fair to say that patents :arrow: inconsistent with open source/free software. I don't know you can take the second step of separating open source/free software, as the author seems to be using them as a package deal. We can't make the leap you make for answer choice (A). Think, for example, about Microsoft Office. Just because it may not be patented (there are several open source and free alternatives) doesn't mean that the Office software becomes automatically free/open source. People still pay for Office; Microsoft didn't release the code at any point. Those who created open source software were just able to engineer it themselves. So even after the patent expires, the specific software doesn't automatically become free/open source. There's nothing in the passage to suggest that's how it would work.

For answer choice (D), we would weaken the argument as described above. Money is a clear motivator for the author of passage B. They specifically state that they are taking an action (patenting their products) for a financial reason. The author links money and motivation himself as part of passage B.

Hope that helps both of you,
Rachael
thanks, Rachal. what really got me to choose A instead of D is the way A was stated, specifically "or less", how less, it might seem ridiculous, but logically speaking it could be a year, a month, or a day. And with D: Less profits still might be sufficient to motivate innovation, innovative people might not put money as the most important factor for doing innovations, and the passage did not state otherwise.

could you please help me straight my thinking?

thanks

Albert
 Adam Tyson
PowerScore Staff
  • PowerScore Staff
  • Posts: 5271
  • Joined: Apr 14, 2011
|
#84254
In a Weaken question, we don't need to prove the argument is wrong, but only need to raise some doubts about it. Answer A does nothing to raise doubts about the claim that patents impede innovation, because even a patent of short duration could still have that effect. Perhaps the impediment of a shorter duration patent is minimal, but it is still, in the eyes of the author, an impediment.

Answer D powerfully challenges that idea by suggesting that patents may actually be a motivator of innovation rather than an impediment. If patented products are more profitable, that gives us a motive to innovate. While some people may not be interested in money, some others may be, and having a motive to innovate weakens the claim that patents impede such innovation. It doesn't disprove the claim, but that's not the goal. It challenges the claim, raising some doubts about it, and that makes D a winner.

Get the most out of your LSAT Prep Plus subscription.

Analyze and track your performance with our Testing and Analytics Package.